subject
Physics, 13.03.2020 19:23 moneymaleia9264

The magnitude J(r) of the current density in a certain cylindrical wire is given as a function of radial distance from the center of the wire's cross section as J(r) = Br, where r is in meters, J is in amperes per square meter, and B = 2.34 × 105 A/m3. This function applies out to the wire's radius of 2.00 mm. How much current is contained within the width of a thin ring concentric with the wire if the ring has a radial width of 13.1 μm and is at a radial distance of 1.50 mm?

ansver
Answers: 1

Another question on Physics

question
Physics, 22.06.2019 04:30
The current in a hair dryer measures 11 amps. the resistance of the hair dryer is 12 ohms. what is the voltage?unit:
Answers: 1
question
Physics, 22.06.2019 07:50
Calculate the ratio of h+ ions to oh– ions at a ph = 6. find the concentration of h+ ions to oh– ions listed in table b of your student guide. then divide the h+ concentration by the oh– concentration. record this calculated ratio in table a of your student guide. compare your approximated and calculated ratios of h+ ions to oh– ions at a ph = 6. are they the same? why or why not? record your explanation in table a. what is the concentration of h+ ions at a ph = 6? mol/l what is the concentration of oh– ions at a ph = 6? mol/l what is the ratio of h+ ions to oh– ions at a ph = 6? : 1
Answers: 1
question
Physics, 22.06.2019 11:30
1. a camcorder has a power rating of 19 watts. if the output voltage from its battery is 7 volts, what current does it use?answer units 2. a 1.5m wire carries a 6 a current when a potential difference of 57 v is applied. what is the resistance of the wire? yourunits 3. a clothes dryer uses about 2 amps of current from a 240 volt line. how much power does it use? yourunits 4.
Answers: 1
question
Physics, 22.06.2019 12:10
Aspring has a natural length of 8 m. if a 12-n force is required to keep it stretched to a length of 10 m, how much work w is required to stretch it from 8 m to 16 m? (round your answer to two decimal places.)
Answers: 1
You know the right answer?
The magnitude J(r) of the current density in a certain cylindrical wire is given as a function of ra...
Questions
question
Mathematics, 12.09.2020 01:01
question
Mathematics, 12.09.2020 01:01
question
History, 12.09.2020 01:01
question
Mathematics, 12.09.2020 01:01
question
Mathematics, 12.09.2020 01:01
question
Mathematics, 12.09.2020 01:01
question
Mathematics, 12.09.2020 01:01
question
Physics, 12.09.2020 01:01
question
Mathematics, 12.09.2020 01:01
question
Arts, 12.09.2020 01:01
question
Mathematics, 12.09.2020 01:01
question
Mathematics, 12.09.2020 01:01
question
Mathematics, 12.09.2020 01:01
question
Mathematics, 12.09.2020 01:01
question
Mathematics, 12.09.2020 01:01
question
Mathematics, 12.09.2020 01:01
question
Mathematics, 12.09.2020 01:01
question
Mathematics, 12.09.2020 01:01
question
Mathematics, 12.09.2020 01:01
question
Mathematics, 12.09.2020 01:01